LSAT and Law School Admissions Forum

Get expert LSAT preparation and law school admissions advice from PowerScore Test Preparation.

 Administrator
PowerScore Staff
  • PowerScore Staff
  • Posts: 8919
  • Joined: Feb 02, 2011
|
#84982
Complete Question Explanation

The correct answer choice is (E).

Answer choice (A):

Answer choice (B):

Answer choice (C):

Answer choice (D):

Answer choice (E): This is the correct answer choice.

This explanation is still in progress. Please post any questions below!
 Haleyeastham
  • Posts: 33
  • Joined: Aug 03, 2015
|
#19487
On number 26, I am confused how E is correct- in the passage, in line 50, the author claims that it is difficult to know how accurate the record is- not IMPOSSIBLE as answer choice E suggests. Can you please explain?

Thanks!
 Steve Stein
PowerScore Staff
  • PowerScore Staff
  • Posts: 1153
  • Joined: Apr 11, 2011
|
#19493
Hi Haley,

That's an interesting question—the author does indeed say that it is difficult to determine how accurately the archaeological record reflects history...and that a hypothesis can only be tested against the best historical data. The author goes on in the final paragraph, though, to say that it's possible some of Lowe's assumptions will be disproven, which implies that ultimately it is impossible to absolutely confirm the accuracy of Lowe's reconstruction.

Tough question! I hope this is helpful—please let me know whether this is clear—thanks!

~Steve
 lunsandy
  • Posts: 61
  • Joined: Oct 14, 2017
|
#41248
Hi Powerscore,

I still don't understand why E is correct. "Impossible to confirm" is that because we do not have the best available data and it says "can be tested ONLY against the best available data." All we have now that we know of is the available evidence from the past (archaeological evidence) so we cannot have an accurate understanding? I also have difficulty in seeing how we jump from "better data might radically alter our understanding from the archaeological evidence" to saying ultimately impossible to confirm the accuracy of the reconstruction.

Would you also eliminate D because of "not an appropriate sources of data." The author here generally accepts John Lowe's claims but questions the accuracy of the data. Thus, "not an appropriate source of data" is too strong of an answer choice?
 Eric Ockert
PowerScore Staff
  • PowerScore Staff
  • Posts: 164
  • Joined: Sep 28, 2011
|
#42042
Hi lunsandy,

I would agree with you that E is not perfect. I think a valid prephrase of the author's position on this issue is that archaeological evidence is an imperfect tool to reconstruct historical activity. Clearly, answer choices (A), (B) & (C) do not reflect this outlook. And you are right that answer choice (D) is too strong. The author does not go so far as to say that using archaeological evidence is "inappropriate."

Answer choice (E) mainly comes out of the sentence at L47-50: "However, it is difficult to know how accurately the archaeological record reflects historic activity..." So from this, while archaeological evidence can be used, it is difficult to know how accurate it is. Answer choice (E) ramps this last part up to "impossible to confirm the accuracy." That seems a bit strong, but considering the alternative answers, this is still the best answer, the one the author is most likely to agree with.

Hope that helps!
 rachelhannah
  • Posts: 9
  • Joined: Jun 23, 2018
|
#48068
I'm struggling with this question. I was between A and E (which are two opposite answers, I know!), and ultimately ended up choosing A, since the author says accuracy can be determined only against the best available data. I reread this last paragraph a few times trying to decide on this question, and ultimately, I decided that the author doesn't rule out that Lowe's hypothesis could at some point be proven accurate. So, I hung my hat on the "can" in answer choice A. This doesn't mean that it's likely, just that it's in the realm of possibility.

Now E, I eliminated because I thought, well...if it's in the realm of possibility that Lowe's hypothesis could be backed up by better data, then E is too strong, stating that it's impossible to confirm the accuracy.

This is a frustrating question for me, for the very reason that I was caught between two opposite answers, and I had trouble picking between them... now "the author would most likely agree" questions are shaking out to be tough for me in general. I don't feel comfortable making that leap in reasoning that I believe Eric made above. I feel better about MSS in LR, because I can better trace the relationship. Here, it feels like I'm picking between something the author believes and something that was explicit... (as in, I feel like the possibility was established for answer choice A, but I do see how the author would believe answer choice E...). Do you have any advice for a better way to read this question? And do you have any advice for just a better way to approach these "Most Likely Agree" questions in general?
User avatar
 Jonathan Evans
PowerScore Staff
  • PowerScore Staff
  • Posts: 726
  • Joined: Jun 09, 2016
|
#48112
Hi, Rachel!

Excellent questions! One issue is that answer choice (A) is a slight misreading of the text. From the text, we know that with additional data, historical hypotheses could be disproved. However, we do not have information in the text to suggest that with additional data, historical hypotheses could be proved.

Thus, you were on the right track, but this slight misunderstanding of the text between lines 44 and 60 led you to eliminate answer choice (E) when in fact (E) was more in sync with the thesis of this part of the passage. This discussion about how additional data could disprove or alter a theory suggests the possibility that new data/information could always alter or disprove existing hypothesis. While the text does not state explicitly that it is impossible to have a complete set of data that would decisively establish the validity of a certain hypothesis, this assumption is in keeping with the author's discussion of the flaw in Lowe's explanation, that is that there could be better data that would disprove it.

In doing these kinds of "author would most likely agree" questions, try to keep in mind the overarching main point/purpose of the passage as an initial process of elimination tool. Use your understanding of the main point/purpose quickly to eliminate two or three losers. When you're left with a couple contenders, match them up with evidence in the passage in order to attempt to eliminate one of them more than prove another one is right.

A problem you had here is that you kinda used (A) to get rid of (E). That's dangerous. Use the text to eliminate answers. Don't use answers to eliminate other answers. In doing so, you run the risk of falling into traps and making unwarranted assumptions. I am confident that you could have caught the information in the text that shows how (A) is unsupported. There is no information in the text that shows that (E) is unsupported. Through this process of elimination, you could have come down to (E) even if you are not completely satisfied with it. Since our job is to find the best answer, you have the skills to do so right now.

Keep up the good work, and keep practicing!
 rachelhannah
  • Posts: 9
  • Joined: Jun 23, 2018
|
#48115
You’re brilliant. :-D

Thank you, Jonathan!

Get the most out of your LSAT Prep Plus subscription.

Analyze and track your performance with our Testing and Analytics Package.